How to prove that every third Fibonacci number is even?

  • Thread starter Thread starter Eclair_de_XII
  • Start date Start date
  • Tags Tags
    even
Click For Summary
The discussion focuses on proving that every third Fibonacci number is even, defined by the recurrence relation F_n = F_{n-2} + F_{n-1}. The user attempts to show that F_n is even if and only if n is divisible by 3 by expressing Fibonacci numbers in terms of mod 2. They illustrate the case for n=3 and propose an inductive step for n=3(k+1), but express uncertainty about the completeness of their proof. A suggestion is made to analyze the Fibonacci sequence in sets of three to clarify the pattern modulo 2. The conversation emphasizes the need for a rigorous inductive proof to establish the claim.
Eclair_de_XII
Messages
1,082
Reaction score
91

Homework Statement


"Consider the sequence ##F_1##, ##F_2##, ##F_3##, . . . , where
##F_1 = 1##, ##F_2 = 1##, ##F_3 = 2##, ##F_4 = 3##, ##F_5 = 5## and ##F_6 = 8##.
The terms of this sequence are called Fibonacci numbers.
(a) Define the sequence of Fibonacci numbers by means of a recurrence relation.
(b) Prove that ##2 | F_n## if and only if ##3 | n##.

Homework Equations


(a) ##F_n=F_{n-2}+F_{n-1}##

The Attempt at a Solution


(b)
Basically, I'm going to express every Fibonacci number in terms of ##mod2## and express ##n## as ##n=3x## for some ##x∈ℕ##.

For ##x=1##, ##F_3=F_1+F_2=1mod2+1mod2=0mod2##.
Then assuming that ##F_{3k}=0mod2## for some ##k>1##, I need to prove that ##F_{3(k+1)}=F_{3k+1}+F_{3k+2}##.

So I have ##F_{3k+1}=0mod2+1mod2## and ##F_{3k+2}=0mod2+1mod2##.
Adding them up gives ##F_{3(k+1)}=F_{3k+1}+F_{3k+2}=0mod2+1mod2+0mod2+1mod2=0mod2##.

And I'm pretty sure this isn't sufficient to complete the inductive proof. Can anyone check my work? Thanks.
 
Physics news on Phys.org
You we're supposed to show iff.

The proof would be simpler if you considered the sequence in sets of 3 and explicitly showed that, modulo 2, the sequence is:

##1, 1, 0,1,1,0 \dots##
 
Question: A clock's minute hand has length 4 and its hour hand has length 3. What is the distance between the tips at the moment when it is increasing most rapidly?(Putnam Exam Question) Answer: Making assumption that both the hands moves at constant angular velocities, the answer is ## \sqrt{7} .## But don't you think this assumption is somewhat doubtful and wrong?

Similar threads

  • · Replies 6 ·
Replies
6
Views
2K
  • · Replies 4 ·
Replies
4
Views
2K
  • · Replies 4 ·
Replies
4
Views
3K
  • · Replies 14 ·
Replies
14
Views
2K
  • · Replies 4 ·
Replies
4
Views
1K
  • · Replies 11 ·
Replies
11
Views
3K
  • · Replies 2 ·
Replies
2
Views
1K
Replies
1
Views
2K
  • · Replies 5 ·
Replies
5
Views
2K
  • · Replies 3 ·
Replies
3
Views
2K